AMBOSS Access Codes
5.21K subscribers
4 photos
40 files
43 links
Follow for Amboss access codes

Be 1st to use it or some one else will use it.
Turn one notification to be 1st to use
Amboss access codes
Download Telegram
Forwarded from Doctopia
A 57-year-old man presents with hematuria and is diagnosed with renal cell carcinoma. Treatment with high-dose interleukin-2 (IL-2) leads to significant tumor regression. What mechanism likely contributed to this outcome?
Anonymous Quiz
14%
Antiangiogenic effect of IL-2
18%
Direct damage to tumor cells by IL-2
43%
IL-2 binding to Fe receptors on cytotoxic lymphocytes
25%
Enhanced activity of natural killer cells
Forwarded from Doctopia
In calculating an aminoglycoside dose for a 58-year-old male with type 2 diabetes, recurrent UTIs, and significant obesity, why would the pharmacy use an adjusted body weight rather than his actual weight?
Anonymous Quiz
18%
Elimination of the drug is predominately by hepatic clearance
36%
Distribution of the drug is limited to the extracellular fluid compartment\
19%
The drug exhibits a high degree of plasma protein binding
27%
The drug is tightly sequestered within adipose tissues
Forwarded from Doctopia
A 40-year-old woman with a history of pulmonary arterial hypertension (PAH) is prescribed sildenafil. Which of the following is the MOST LIKELY mechanism of action of sildenafil in PAH?
Anonymous Quiz
7%
Endothelin receptor antagonism
70%
Inhibition of phosphodiesterase-5 (PDE-5)
4%
Prostacyclin antagonism
18%
Vasodilation via antagonism of alpha-adrenergic receptors
We have create a website to post Uworld and Amboss and USMLErx Quetions and Topic based questions in our website so you can test your self free of cost https://doctopia.in/login . which system do you guys want us to start with ?? comment down below
Sign up today so when we post the link you can just click on it and get redirect to the qbank
AMBOSS Access Codes
https://doctopia.in/tests/start?id=660bb476db4ec57cdac0acbe&name=Pharmacology
Uworld usmle step 1 Respiratory Pharm H1 Blockers Quetions has been uploaded Test yourself
Sign up and select USMLE Step 1
Click on Qbnak and select Respiratory
Then click and H1 Blockers and start the test
This quetions are grabed from UWORLD USMLE RX and Amboss Combined More Topics will be uploaded soon
Who ever Scored 4/4 Please Share a Screenshot in the comments and get a free Amboss Account

Comment here 👇
Patient with inflammatory bowel disease (IBD) presents with recurrent urinary tract infections and pneumaturia. Which of the following is the most likely explanation?
Anonymous Quiz
53%
Enterovesical fistula
24%
Pyelonephritis
16%
Interstitial cystitis
6%
Nephrolithiasis
A 70-year-old man presents with exertional dyspnea, angina, and syncope. Auscultation reveals a harsh systolic murmur at the right 2nd intercostal space, radiating to the carotids. What is the most likely diagnosis?
Anonymous Quiz
15%
Mitral regurgitation
62%
Aortic stenosis
8%
Mitral stenosis
15%
Aortic regurgitation
A 50-year-old with a history of MI presents with exertional dyspnea and fatigue. A holosystolic, high-pitched murmur is heard at the apex, radiating to the axilla. What is the most likely diagnosis?
Anonymous Quiz
12%
Aortic stenosis
24%
Mitral stenosis
59%
Mitral regurgitation
5%
Tricuspid regurgitation
Forwarded from Doctopia
Achalasia.pdf
6 MB
AYNK 06 : Achalasia
Forwarded from Doctopia
Marfan Syndrome.pdf
6.8 MB
AYNK 07 : Marfan Syndrome
Forwarded from Doctopia
Marfan Syndrome.pdf
6.8 MB
AYNK 07 : Marfan Syndrome
Forwarded from Doctopia
Join waitlist be the first to use Doctopia for free 🤩

👇👇👇

https://doctopia.in/